You are on page 1of 82

Welcome sunil.

andhare01
Your Test Results
Back

Test Summary
Marks Rank
-0.5 59

Test POLITY-03
Subject GS
Test Duration 01:00:00
Max. Marks 100.00
Correct Answers 0
Wrong Answers 0
Attempted/Not Attempted Count Attempted : 0
Not Attempted : 100
Positive Marks 0.00
Negative Marks 0
Obtained Marks 0
Percentage 0%

Test Information in Detail


Question No. 1
Question

खालील वधाने वाचा आ ण दले यांपैक यो य पयाय नवडा.

(अ)    अमे रकन सव च यायालय ‘काय ा या यो य येनुसार’ नाग रकां या ह कांचे संर ण करते.

(ब)     भारतीय सव च यायालय ‘काय ाने था पत केले या येनुसार’ नाग रकां या ह कांचे संर ण करते.

(क)     भारतीय सव च यायालयाला स लागारी अ धकार े नसून अमे रकन सव च यायालयाला असे स लागारी े आहे.

(ड)    अमे रकन सव च यायालया या पुन वचारा या अ धकार े ात फ घटना मक खटले येतात.

Options

a   वधान क अस य आहे.
b   वधाने ब आ ण क ही अस य आहेत
c   वधान ड हे अस य आहेत
d  वरीलपैक सव वधाने स य आहेत.

Your Answer unanswered

Correct Answer a
Explanation
 अमे रके या व भारता या यायालयांची तुलना.
भारतीय सव च यायालय अमे रक सव च यायालय
यांचे ांर भक े संघरा यीय खट यापुरते
फ संघरा यीय खट यांबाबत ारं भक
मया दत नसून यामधे ना वक दल, सागरी
अ धकार दे ते.
वहार, राज त इ.चा समावेश.
या या पुन वचारा या अ धकार े ात फ
याला स लागारी े आहे.
घटना मक खटले येतात.
या यक पुन वलोकनाची ा ती खूप
या यक पुन वलोकनाची ा ती मया दत.
व तृत.
कायदय्◌ाने था पत केले या यानुसार काय ा या यो य येनुसार हे
हे नाग रकां या ह कांचे संर ण करते. नाग रकां या ह कांचे संर ण करते.
i.e- Process establised by Law i.e Due Process of Law.
Question No. 2
Question

पुढ लपैक कोणती/ते वधान/ने यो य आहे/त ?  

(अ)     महारा रा यात आजपयत तीन वेळा रा पती राजवट लागू झा या आहेत.

(ब)     1978 म ये पुलोदचे सरकार बरखा त झा यामुळे प ह यांदा रा पती राजवट लागू झाली होती.

(क)     2014 म ये रा वाद काँ ेस प ाने पा ठबा काढू न घेत याने रा यात रा पती राजवट लागू झाली होती.

(ड)    रा यांम ये रा पती राजवट अनु छे द 352 नुसार लावली जाते.

Options

a  फ अ, ब आ ण क
b  फ अ आ ण क
c फ अ, क आ ण ड  
d  वरीलपैक सव

Your Answer unanswered

Correct Answer b
Explanation
 रा यांमधे रा पती राजवट लाव याचे 2 कार -
(1) अनु छे द 356 (रा यात घटना मक यं णा कोलमड यास)
(2) अनु छे द 365 (क ाने दले या आदे शाचे पालन न के यास)

Question No. 3
Question
 खालील (अ) आ ण (ब) वधाने वाचून पयायी उ रातील यो य पयाय नवडा.

वधान (अ) भारतीय नधम पणाला एक‘‘मोझेक’’ हटले आहे.

वधान (ब) भारतात कोण याही धमाला ‘रा य धमाच’◌े थान नाही.

Options

a अ अ◌ा ण ब दो ही वधाने स य असून ब हे अ ची यो य कारणमीमांसा दे त.े  


b  अ अ◌ा ण ब दो ही वधाने स य असून ब हे अ ची यो य कारणमीमांसा दे ते नाही.
c  अ वधान स य आहे परंतु ब चूक आहे.
d  अ वधान चूक आहे परंतु ब स य आहे.

Your Answer unanswered

Correct Answer a
Explanation
भारतीय नधम पणा हणजे मोझेक. 
          कारण
         (1)      यात अनेक धम व भाषा मानाने व वातं याने राहतात.
         (2)    सव गट आपली ओळख न न करता एक सं म सं कृतीचा घटक होतात.
         (3)    सामा य माणसाला ते एक आणते.
        (4)    कोण याही धमाला रा यधमाचेे थान नाही (सवधमसमभाव).

Question No. 4
Question
यो य जो ा जुळवा.

तंभ (अ) अनु छे द                                                तंभ (ब)तरतूद

(अ)   अनु छे द 243Z-D                        (i) यायालयीन ह त ेपास तबंध.

(ब)    अनु छे द 243 -I                              (ii) लेखा परी ण

(क)   अनु छे द 243 -J                            (iii) रा य व अ◌ायाेग. 

(ड)    अनु छे द 243- O                           (iv) ज हा नयोजन आ ण वकास स मती थापन करणे.

Options

a  अ-iv, ब-ii, क-iii, ड-i


b  अ-i, ब-iv, क-iii, ड-ii
c  अ-iv, ब-iii, क-ii, ड-i
d  अ-iv, ब-i, क-ii, ड-iii

Your Answer unanswered

Correct Answer a
Explanation
 अनु छे द - 243 (A) = ामसभा (2) 243 (B) - पंचायतीची थापना.
(3) 243 (C) - पंचायत ची संरचना (4) 243 (D) - आर त जागा.
(5) 243 (E) - पंचायत सं थांचा कायकाळ (6) 243 (F) सद य अपा ता
(7) 243 (I) - रा य व अ◌ायोग (8) 243 (K) नवडणूक अ◌ायोग
(9) 243 (J) - लेखा परी ण (10) 243 (L) क. दे शातील पंचायती सं था.
(11) 243 (M) - व श दे शाबाबत पंचायती तबंध
(12) 243 (O) यायालयीन ह त ेपास तबंध
(13) 243 (P) - ज हा नयोजन आ ण वकास आघाडी.

Question No. 5
Question

 ‘‘Doctrine of Severability’’ हणजे ....... ?

Options

a संसदे ने केले या कायदय्◌ाचे पुन वलोकन करणे.  


b एखादा कायदा जेव ा माणात वसंगत आहे तेव ाच माणात तो शू यवत कर याचे त व ान होय. 
c  एखादा कायदा जनतेला लागू कर यास सरकारला आदे श दे णे.
d  जन हतासाठ आव यक असलेला कायदा बनव यासाठ सरकारला सूचना दे ण.े

Your Answer unanswered

Correct Answer b
Explanation
 Doctrine Severability हणजे एखादा कायदा जेवढया माणात वसंगत आहे तेव ाच माणात तो शू यवत घो षत
कर याचे त व ान होय.

Question No. 6
Question

खालीलपैक कोणते अ धकार सव च यायालया या अ धकारांगत अंतभूत नाहीत ?

(अ)     रा पती .ि◌ंकंवा उपरा पती नवडणूक नकालाबद्दल ववाद अस यास सु ीम कोट नकाल दे ते हा अ धकार ारं भक/
असमावेशक/अं तम असेल.

(ब)     UPSC अ य कवा सद य यां यावरील दोषारोपणाची चौकशी करणे.जर ते दोषी असतील तर रा पत ना पदाव न र
कर याची शफारस करतात, जे रा पतीवर बंधनकारक असते.

(क)    सु ीम कोट वत: या नकालाचे वत:च पुन वलोकन क शकते. सव च यायालयाचे सव नकाल इतर सव यायालयांवर
बंधनकारक असतात परंतु वत : वर मा बंधनकारक नसतात.

Options

a  फ ब
b फ क 
c  ब आ ण क
d  यांपैक नाही.

Your Answer unanswered

Correct Answer d
Explanation
 वरीलपैक सव अ धकार सव च यायालया या अ धकारांगत अंतभूत आहेत.
Question No. 7
Question

यो य जो ा जुळवा.

            तंभ (अ) वषय                                                                                        तंभ (ब)घटना ती

(अ)    रा य तरीय भाडे याया धकरणांची 

थापना                                                                        (i) 69 वी घटना ती अ ध नयम,1991.

(ब)    अ णाचल दे शम ये पंचायत म ये  

अनुसू चत जात ंसाठ आर ण नाही.                             (ii) 75 वी घटना ती अ ध नयम,1994.                          

(क)    ाम .ि◌ंकंवा इतर था नक 

पातळ वर पंचायतीची थापना                                      (iii) 83 वी घटना ती अ ध नयम,2000.                           

(ड)    द लीला रा ीय राजधानी 

दे शाचा  दजा दे यात आला.                                        (iv) 73 वी घटना ती अ ध नयम,1992.                                                   


                         

Options

a  अ-iv, ब-iii, क-ii, ड-i


b  अ-iii, ब-ii, क-iv, ड-i
c  अ-ii, ब-iii, क-iv, ड-i
d  अ-iv, ब-iii, क-i, ड-ii

Your Answer unanswered

Correct Answer c
Explanation
 

Question No. 8
Question

 यो य पयाय नवडा.


वधान (A) यायालया या आदे शांचे हेतूपूवक उ लंघन कवा यांचे पालन न करणे व यायालयां या वहारांबददल अपश द वापरणे हा
यायालयाचा अवमान आहे.
वधान (R) अवमानकारक वागणुक साठ श ा कर याचे अ धकार द या शवाय यायालयीन कृतीशीलता य ात आणता येणार
नाही.

Options

a  दो ही वधाने बरोबर असून R हे वधान A चे यो य प ीकरण दे त.े


b  दो ही वधाने बरोबर असून R हे वधान A चे यो य प ीकरण दे त नाही.
c   वधान A बरोबर आहे, पण वधान R चूक आहे.
d   वधान A चूक आहे, पण वधान R बरोबर आहे.

Your Answer unanswered

Correct Answer b
Explanation
 सव च यायालय या या अवमान करणी दं ड .ि◌कंवा श ा अनु छे द 129 नुसार दे ते हणजेच या अनु छे दानुसार याला
अ भलेख यायालयाचे अ धकार ा त होतात.

Question No. 9
Question

 ‘‘परक य आ मपणापासून व अंतगत अशांततेपासून येक रा याचे संर ण करणे हे क ाचे कत आहे’’ अशी तरतूद भारतीय
रा यघटने या कोण या अनु छे दात केली आहे ?

Options

a  अनु छे द 215
b  अनु छे द 275
c  अनु छे द 325
d  अनु छे द 355

Your Answer unanswered

Correct Answer d
Explanation
 (1) अनु छे द 355 - परक य आ मण आ ण अंतगत अशांतता यांपासून रा यांचे र ण कर याचे क ाचे कत .
(2) अनु छे द - 325 धम, वंश, जात कवा लग या कारणाव न कोण याही ला मतदार याद त समावेश कर यास अपा
ठर वले जाणार नाही व वशेष मतदार याद त समावेश कर याची मागणी करता येणार नाही.
(3) अनु छे द 275 - क ाकडू न व श रा यांना अनुदान.
(4) अनु छे द 215 - उ च यायालये ही अ भलेख यायालये असतील. 

Question No. 10
Question

 खालीलपैक अचूक वधान/ने काेणते/ती ?


वधान (अ) घटना मक सरकार हणजे जे रा या या अ धकारा या हतासाठ वातं यावर भावी नयं णे घालते.
वधान (ब) घटना मक सरकार हणजे जे वातं या या हतासाठ रा या या अ धकारांवर भावी नयं णे घालते.

Options

a  फ वधान अ अचूक आहे.


b  फ वधान ब अचूक आहे.
c   वधान अ आ ण ब दो ही अचूक अ◌ाहेत.
d   वधान अ आ ण ब दो ही अचूक नाहीत.

Your Answer unanswered

Correct Answer b
Explanation
 घटना मक सरकार हणजे जे सरकार रा या या हताचा वचार करते तसेच रा यातील या वातं या या
हतासाठ रा या या अ धकारावर भावी नयं ण ठे वतेे.

Question No. 11
Question
खालील वधाने वचारात या.

(अ)    आंतररा यीय लवादाने यां याकडे आले या ववादाची पूण तपासणी क न दोन वषा या आत आपला नणय दे णे अपे त
असते.

(ब)   लवादाला नणय दे यासाठ उशीर झा यास क सरकार लवादास एकावेळ मह म एक वष वाढवून दे ऊ शकते.

Options

a   वधान अ यो य असून, ब अयो य आहे.


b   वधान ब यो य असून अ अयो य आहे.
c  दो ही अ आ ण ब वधाने यो य आहेत.
d  दो ही अ आ ण ब वधाने अयो य आहेत.

Your Answer unanswered

Correct Answer d
Explanation
अनु छे द = 262 आंतररा यीय जल ववादा या नवाड् याची तरतूद.
अनु छे द = 263 आंतररा यीय प रषदा थापन करणे.
आंतरा यीय लवादाने या याकडे आले या ववादाची पूण तपासणी क न एका वषा या आत आपला नणय दे णे अपे त
असते.
असा नणय न झा यास क सरकार लवादाला एका वेळ मह म 6 म हने वाढवून दे ते.
 

Question No. 12
Question
यो य जो ा जुळवा.

तंभ (अ) तंभ                                                  (ब)नद ववाद याया धकारण थापना वष

(अ) रावी व बयास                                                                               (i) 1986.

(ब) कावेरी                                                                                          (ii) 1990

(क) कृ णा                                                                                         (iii) 2004 

(ड) वसुंधरा                                                                                         (iv) 2010

Options

a  अ-i, ब-ii, क-iii, ड-iv


b  अ-iv, ब-iii, क-ii, ड-i
c  अ-iv, ब-ii, क-iii, ड-i
d  अ-i, ब-iii, क-ii, ड-iv

Your Answer unanswered

Correct Answer a
Explanation
 (1) रावी व बयास जल ववाद /नद - पंजाब, ह रयाणा, राज थान.
(2) कावेरी नद जल ववाद - कनाटक, केरळ, त मळनाडू व पु चेरी.
(3) कृ णा नद जल ववाद - महारा , कनाटक, आं दे श.
(4) महानद जल ववाद - गोवा, कनाटक, महारा .

Question No. 13
Question
 अंदाज स मतीबाबत खालील वधाने वचारात या.

(अ)    वातं यानंतर त कालीन अथमं ी जॉन मथाई यां या शफारशीनुसार या स मतीची थापना 1950 साली कर यात आली.

(ब)      सु वातीला या स मतीची सद यसं या 25 होती.

(क)    या स मतीतील सव सद यं◌ाचा कालावधी 1 वषाचा असतो.

(ड)    या स मतीला भारताचे नयं क व लेखापरी कांचे सहकाय मळत नाही.

वरीलपैक अचूक वधान/ने कोणती? 

Options

a  अ,ब आ ण ड
b  ब, क आ ण ड
c  ब आ ण क
d  वरीलपैक सव

Your Answer unanswered

Correct Answer d
Explanation
 अंदाज स मती - थापना 1950
जॉन मथाई यां या शफारशीनुसार.
सु वातीला सद य सं या = 25 .
सव सद यांचा कालावधी 1 वष असतो.

Question No. 14
Question
यो य जो ा जुळवा.

      तंभ (अ)                                                                                  तंभ (ब) तरतुद अनु छे द

(अ) ाथ मक तरावर मातृभाषेत                               (i) अनु छे द 350(A) श ण दे याची सोय

(ब) भा षक अ पसं याकांसाठ                                    (ii) अनु छे द 350(B) वशेष अ धकारी

(क) हद भाषे या वकासासाठ                                    (iii) अनु छे द 347 आदे श.

(ड) रा यातील काही लोक बोलत असले या             (iv) अनु छे द 351 भाषे वषयी वशेष तरतुद . 

Options

a  अ-i, ब-ii, क-iii, ड-iv


b  अ-iii, ब-iv, क-ii, ड-i
c  अ-i, ब-ii, क-iv, ड-iii
d  अ-iv, ब-iii, क-ii, ड-i

Your Answer unanswered

Correct Answer c
Explanation
 

Question No. 15
Question
73 ा घटना ती संदभात खालील वधाने वचारात या.

(अ) ामसभेची न मती करावी.

(ब) 33% जागा यांसाठ राखीव ठे वा ात .

(क) पंचायतीला वाय सं था हणून काम कर यास अ धकार दे ण.े

(ड) आमदार व खासदारांना या या काय े ातील येणा या त ही तरावर पंचायतम ये त न ध व दे ण.े (इ) इतर मागासवग यासाठ
(OBC) आर ण.

(ई) रा य नवडणूक आयोगाची थापना.

वरील तरतुद ना ‘‘ऐ छक तरतुद ’’आ ण‘‘बंधनकारक तरतुद ’’अशा दोन गटांत वभागणी केली असता, याबाबत यो य भा य करणारा
पयाय नवडा.

Options

a बंधनकारक तरतुद          ऐ छक तरतूद


अ, ब, क, ड,                              इ, ई
b  बंधनकारक तरतुद          ऐ छक तरतूद
     ब, क, ड,                                   अ, इ, ई
c  बंधनकारक तरतुद          ऐ छक तरतूद
   अ, ब, ई,                                          क, ड, इ
d  बंधनकारक तरतुद          ऐ छक तरतूद
    अ, ब, इ, ई,                                क, ड

Your Answer unanswered

Correct Answer c
Explanation
 3 ा घटना ती कायदय्◌ाने पंचायत राज या तरतुद चे दाेन भाग केले आहेत.
 
अ नवाय तरतुद (भाग-1) ऐ छक तरतुद (भाग - 2)
गावासाठ कवा गावा या गटाक रता संसद सद य व रा य वधानमंडळ सद य यांना यां या
ामसभेची थापना मतदारसंघातील त न ध व दे णे.
तरीय तरांवर पंचायतीची थापना कोण याही पातळ वर पंचायतीमधे OBC साठ सद य
करावी. तसेच अ य पदासाठ जागा आर त ठे वणे.
पंयायत ना वरा यसं था हणून कामे पार पाडणे श य
त ही तरांवरील पंचायतीमधील
हावे या ीने यांना आव यक असतील असे अ धकार व
जागांक रता य नवडणूक यावी.
ा धकार धान करणे.
पंचायत नवडणुक साठ कमान 21
 
वष वय अट ठे वली.
दर 5 वषानी व आयोग थापणे.  
रा य नवडणूक अ◌ायोग.  
त ही तरांवर सद य तसेच
अ य पदा या 1/3 जागा  
म हलांसाठ राखीव ठे वणे.

Question No. 16
Question

खालील वधानांचा वचार करा.

(अ)    73 ा घटना तीनुसार अनु छे द 243(K) हा पंचायती या नवडणुका घे या वषयी तरतूद करतो.

(ब)      पंचायती या सव नवडणुकांसाठ मतदार या ा तयार करणे आ ण नवडणूक घेणे हे रा य नवडणूक आयोगाचे काय आहे.

(क)     रा य नवडणूक आयोगा या आयु ांची नेमणूक आ ण यांना पदाव न र कर याचे अ धकार रा यपालांना आहेत.

वरील वधानापैक अचूक वधान/ ने कोणती/ते ?

Options

a  फ अ आ ण ब 
b  फ ब आ ण क
c  फ अ आ ण क
d  वरीलपैक सव

Your Answer unanswered

Correct Answer a
Explanation
 रा य नवडणूक आयोगा या आयु ांना पदाव न र कर याचा अ धकार हा रा यपालांना नसून रा पत ना आहे.

Question No. 17
Question

नाग रक व कायदा 1955 अंतगत खालीलपैक कोण या पयाया दारे एखाद भारताचा नाग रक बनू शकते ?

(अ)    भूमी अ ध हीत झा यामुळे

(ब)    वारसा ह कांने

(क)    ज म भारतात झा याने

(ड)    रा ीयीकरणा दारे 

(इ)   नादणी दारे

Options

a  ब, क, ड, इ
b  अ, ब, क, ड
c  अ, क, ड, इ
d  अ, ब, क, इ

Your Answer unanswered

Correct Answer d
Explanation
 नाग रक व कायदा 1955
कायदयात 5 कारचे नाग रक व
(1) ज माने (2) वंश वाने (3) न दणीकृत (4) नाग रक करणाने (5) भू दे शा या एक करणा दारे.
सु वातीला 1955 या काय ात रा कुला या नाग रक वाची तरतूद होती पण 2003 या Amendment दारे र
कर यात आली.

Question No. 18
Question

42 वी घटना ती 1976 ने कोणती ‘‘मागदशक त वे’’ (अनु छे द) घटनेत समा व केली ?

(अ)   अनु छे द 38(2)

(ब)   अनु छे द 43(A).

(क)   अनु छे द 48(A).

(ड)   अनु छे द 39.

(इ)   अनु छे द 39(A).

(ई)   अनु छे द 43(B).

Options

a  अ, ब, ड, ई
b  क, ड, इ, ई
c  अ, ब, क, इ
d  ब, क, ड, इ

Your Answer unanswered

Correct Answer d
Explanation
42 ा घटना तीने खालील मागदशक त वे समा व केली.
कलम 39 (A) , कलम 39
कलम 43 (A), कलम 48 (A)
44 ा Amendment दारे मागदशक त वे समा व - कलम 38 (2)

Question No. 19
Question
यो य जोडया जुळवा.

          तंभ (अ)                                                                                                                  तंभ (ब)

           तरतुद                                                                                                                      अनु छे द

(अ) अ धकृत भाषेसंबंधी संसदे ची स मती                                                       (i) अनु छे द 345

(ब) रा यारा यांम ये वा रा य-क ामधील भाषा                                          (ii) अनु छे द 347

(क) एखा ा रा यातील व श भा षक                                                       (iii) अनु छे द 344 गटांसाठ तरतूद.

(ड) रा यासाठ अ धकृत भाषा                                                                            (iv) अनु छे द 346

Options

a  अ-iv, ब-iii, क-ii, ड-i


b  अ-iv, ब-ii, क-i, ड-iii
c  अ-iii, ब-ii, क-iv, ड-i
d  अ-iii, ब-iv, क-ii, ड-i

Your Answer unanswered

Correct Answer d
Explanation
 

Question No. 20
Question
शू य हरा या संदभात खालीलपैक कोणती वधाने अचूक आहेत ?

(अ)   संसद य कायप दतीम ये शू य हर ही नावी यपूण संक पना भारतात उदयाला आली.

(ब)    ही संक पना 1965 पासून य ात आली.

(क)    शू य हर हे संसद सद यांना उपल ध असलेले अनौपचा रक व पाचे मा यम आहे.

(ड)      ो राचा तास आ ण सभागृहाचे कामकाज यां या मध या काळाला शू य हर हणतात.

Options

a  फ अ, ब, क
b  फ अ, क, ड
c  फ ब व क
d  वरील पैक सव

Your Answer unanswered

Correct Answer b
Explanation
 संसद य कायपद्धतीत शू य हर ही संक पना भारतात उदयास आली.
 
 

Question No. 21
Question
खालील वधाने वचारात या.

(अ)   लोकायु सं था थापन करणारे महारा हे भारतातील प हले रा य आहे.

(ब)    लोकायु सं थेची थापना थम महारा ात 1973 म ये झाली.

(क)    लोकायु सं थेबाबत कायदा ओ डशाने 1970 म ये सव थम केला.

(ड)    कनाटक लोकायु सं था संपूण दे शात सवात जा त भावशाली हणून ओळखली जाते.

वरीलपैक बनचूक वधान/ने हणून कोणती/ते ?

Options

a  फ अ, ब, क
b  फ ब, ड
c  फ अ, क
d  फ अ, क, ड

Your Answer unanswered

Correct Answer d
Explanation
 लाेकपाल सं था थापन करणारे महारा हे भारतातील थम रा य अ◌ाहे.
महारा लाेकपाल सं था थापना 1971 साली झाली.
स या भारतातील एकूण 22 रा यांमधे लाेकायु पदाची तरतूद झालेली आहे.

Question No. 22
Question
खालील वधाने वचारात या.

(अ)      श ची तरतूद करणारे वधेयक हे धन वधेयक असेलच असे नाही.

(ब)      एक त नधीतून खचा वषयीचे वधेयक हे धन वधेयक असेलच असे नाही.

(क)   भारतीय रा यघटनेत व वधेयकाचे चार कार मांडले आहेत.

वरीलपैक कोणती/ते वधान/ने बरोबर आहेत ?

Options

a  अ आ ण ब
b  ब  आ ण  क
c  अ आ ण क
d  अ, ब, क

Your Answer unanswered

Correct Answer a
Explanation
 भारतीय रा यघटनेत व वधेयकाचे 3 कार मांडलेले आहेत.

Question No. 23
Question
यो य जो ा जुळवा.

          तंभ (अ)                                                                          तंभ (ब) 

       सं था दोन                                                                       सभांतील अंतर

(अ)   पंचायत स मती                                             (i) तीन म ह यांपे ा जा त नाही.

(ब)    ज हा प रषद                                                (ii) येक म ह यात एक

(क)    ामपंचायत                                                  (iii) एक म ह यापे ा जा त नाही.

(ड)    ामसभा                                                         (iv) चार म ह यापे ा जा त नाही. 

Options

a  अ-iii, ब-i, क-ii, ड-iv


b  अ-iii, ब-ii, क-i, ड-iv
c  अ-iii, ब-iv, क-ii, ड-i
d  अ-iii, ब-iv, क-i, ड-ii

Your Answer unanswered

Correct Answer a
Explanation
 

Question No. 24
Question

103 वी घटना ती काय ाबाबत खालीलपैक कोणते वधान चुक चे आहे ?

(अ)    हा घटना ती कायदा 12 जानेवारी 2019 पासून अंमलात आला.

(ब)     हा कायदा अनु छे द 14,15 आ ण 16 म ये ती करतो.

(क)     हा कायदा भारतीय रा यघटनेतील मागदशक त वे (DPSP) अनु छे द 45 ला अनुस न कर यात                             आला
आहे.
Options

a  फ अ
b  फ ब व क
c  फ क
d  एकही नाही

Your Answer unanswered

Correct Answer d
Explanation
 103 वी Amendment - अंमल - 12 जानेवारी 2019
कलम 14, 15, 16 मधे ती सुच वतो.
मागदशक त वातील कलम 45 ला अनुस न कर यात आला आहे.

Question No. 25
Question

पुढ ल वधाने वचारात या.

(अ)    भारतीय रा यघटनेतील अनु छे द 352 हे रा ीय आणीबाणी बाबत तरतुद करते.

(ब)     रा ीय आणीबाणी म ये रा याचे व धमंडळ बरखा त केले जाते.

(क)     रा ीय आणीबाणीची घोषणा झा यापासून एका म ह या या आत दो ही सभागृहांची वशेष ब मताने मा यता आव यक असते.

(ड)     संसदे या दो ही सभागृहांनी मा यता द यास अशी आणीबाणी एका वषासाठ लागू राहते आ ण पु हा कालावधी वाढवायचा
अस याचा न ाने ठराव मांडावा लागतो, असा ठराव अ न त वेळा मांडला जाऊ शकतो.

आता सांगा क ,

Options

a   वधाने अ, ब, क बरोबर तर ड चूक आहेत.


b   वधाने अ, क बरोबर तर ब, ड चूक आहेत.
c   वधाने अ, क, ड बरोबर तर ब चूक आहेत.
d   वधाने अ, ब, क, ड सव बरोबर आहेत.
Your Answer unanswered

Correct Answer b
Explanation
 अनु छे द 352 - रा ीय आणीबाणीसंबं धत तरतुद .
रा ीय आणीबाणीत रा यांची व धमंडळे बरखा त होत नाहीत.

Question No. 26
Question

 ज मूका मीर रा याचे पुनरचना वधेयक जे संसदे ने मंजूर केले ते भारतीय रा यघटनेतील अनु छे द 3 या आधारावर मांडले होते.
याबाबत पुढ ल वधाने वचारात या. 

(अ)    असे वधेयक लोकसभा आ ण रा यसभा वशेष ब मताने मंजूर करते.

(ब)      असे वधेयक संसदे त मांड यापूव रा पत ची पूवसंमती आव यक असते.

(क)      रा पती संमती दे यापूव असे वधेयक संबं धत रा या या व धमंडळाकडे रा याचा कोन मांड यासाठ पाठवतात.

(ड)     अ या वधेयकावर रा याने केले या शफारशी संसदे ने वीकारणे बंधनकारक असते.

वरीलपैक कोणते/ती वधान/ने अचूक नाहीत ?

Options

a  फ बआणड
b  फ अआणक
c  फ अआणड
d  फ ड

Your Answer unanswered

Correct Answer c
Explanation
असे वधेयक लाेकसभा आ ण रा यसभा सा या ब मताने मंजूर करते.
अशा वधेयकावर रा याने केले या शफारशी संसदे ने वीकारणे बंधनकारक नसते.
 

Question No. 27
Question

 ‘‘कॅ बनेटची कूमशाही’’ असे कोण या दे शा या कॅ बनेटचे वणन ‘‘रॅ से मूर’’ यांनी केलेले आहे ?

Options

a   इं लंड
b  भारत
c  चीन
d  अमे रका

Your Answer unanswered

Correct Answer a
Explanation
 

Question No. 28
Question
खालील वधानांचा वचार करा.

(अ)    भारत आ ण टनम ये रा पती हे संसदे चा अ वभा य भाग असतात.

(ब)     अमे रकेम ये रा ा य हे कायदे मंडळाचे अ वभा य भाग नसतात.

(क)     भारत व टनम ये कायकारी मंडळ हे कायदे मंडळाचा भाग असते.

(ड)     अमे रकेम ये मा कायकारी मंडळ हे कायदे मंडळाचा भाग नसते.

वरीलपैक अयो य नसणारे वधान कोणते ?

Options

a  फ अ आ ण क
b फ बआणड 
c  फ अ, क, ड 
d  अ, ब, क, ड

Your Answer unanswered

Correct Answer d
Explanation
 सव वधाने यो य आहेत.

Question No. 29
Question
खालीलपैक काेण या प र थतीत लं बत वधेयके पगत होत नाहीत ?

(अ)    रा पत या संमतीसाठ लं बत असणारी.

(ब)     रा पत नी फेर वचाराथ पाठ वलेली.

(क)   असे वधेयक, जे मतभेदामुळे रा पत नी वसजनापूव संयु बैठक ची अ धसूचना जारी केलेली अस यास.

(ड)    रा यसभेने संमत केलेले परंतु लोकसभेत लं बत असलेले वधेयक

Options

a  फ अ, ब, क
b  फ अ, ब, ड
c  फकत अ, क, ड
d  फ अ, ब, क, ड

Your Answer unanswered

Correct Answer d
Explanation
 

Question No. 30
Question

संसद सभागृहा या अवमानाबददल श ा कवा तु ं गवास दे याचा नणय सव च यायालयाने खालीलपैक कोण या खट यात दला
?
 

Options

a  केशवानंद भारती खटला


b  सचलाईट खटला
c  केशव सग खटला
d  गोलकनाथ खटला

Your Answer unanswered


Correct Answer c
Explanation
 संसद सभागृहा या अवमानाबददल श ा/दं ड दे याचा नणय
सव च यायालयाने केसवा सग खट यात दला आहे.

Question No. 31
Question

पुढ ल दोन वधानांचा वचार करा.

(अ)     1950 साली जॉन मथाई यां या शफारशीनुसार सावज नक उप म स मतीची थापना कर यात आली आहे.

(ब)      1964 साली कृ ण मेनन या शफारशीनुसार लाेकअंदाज स मतीची थापना कर यात आली.

कोणते वा य पूणपणे बरोबर आहे ?

Options

a  फ अ 
b  फ ब 
c  फ अ व ब
d  अ व ब दो ही नाही.

Your Answer unanswered

Correct Answer d
Explanation
 जॉन मथाई यां या शफारशीनुसार लाेक अंदाज स मतीची थापना. कृ णमेनन यां या शफारशीनुसार सावज नक उप म
स मतीची थापना.

Question No. 32
Question
तंभ मांक I ची याद ही तंभ मांक II शी जुळणी करा व खाली दले या पयायामधून बरोबर सांके तकांची नवड करा.

                        तंभ (I)                                                                                                                    तंभ (II)

(अ)     सभागृहाचे ल आक मकपणे                                                                              (i)    नदा ंजक ठराव

    उदभवले या सावज नक मह वा या  ाकडे वेधून घेण.े

(ब)    मं मंडळावर अ व ास दश वणे                                                                               (ii)    थगन ताव

(क)    कोण याही मु यावर चचबाबत कपात                                                                      (iii)    ठराव

(ड)      व श मु यावर सभागृहाला मत                                                                          (iv)      प रसमा ती ताव                         


                                            कर यास भाग  पाडणे.

Options

a  अ-ii, ब-iv, क-iii, ड-i


b  अ-ii, ब-i, क-iv, ड-iii
c  अ-i, ब-ii, क-iii, ड-iv
d  अ-i, ब-iii, क-ii, ड-iv

Your Answer unanswered

Correct Answer b
Explanation
 

Question No. 33
Question
सव च यायालयाची खालीलपैक ारं भक अ धकार े े कोणती आहेत ?

(अ) रा या - रा यांमधील वाद

(ब) क - रा यांमधील वाद

(क) अनेक रा यां व द क सरकारमधील वाद 

(ड) व आयोगासंबंधी वाद

Options

a  फ अ, ब, ड
b  फ बवक
c  फ अ, ब, क
d  फ बवड

Your Answer unanswered

Correct Answer c
Explanation
 सव च यायालयाची ारं भक े .े
(1) रा यारा यांतील वाद.
(2) क आ ण रा यातील वाद
(3) अनेक रा य व क सरकारमधील वाद

Question No. 34
Question
पुढ ल वधाने वचारात या. 

(अ)      भारतीय रा यघटनेनुसार संबं धत रा य मं मंडळाने राजीनामा दला तर महा धव ा यांनी सु दा आप या पदाचा राजीनामा
दे याची तरतूद आहे.

(ब)      महा धव यांचे पगार आ ण भ े घटनेने न त केलेले नाहीत.

(क) महा धव ा हे रा यपालां या मज ने पद धारण करतात.

आता सांगा क ,

Options

a  फ वधाने अ आ ण क बरोबर आहेत.


b  फ वधाने क बरोबर आहेत.
c  फ वधाने ब आ ण क बरोबर आहेत.
d  फ वधाने अ आ ण ब बरोबर आहेत.

Your Answer unanswered

Correct Answer c
Explanation
 भारतीय रा यघटनेनुसार सबं धत रा य मं मंडळाने जर राजीनामा दला तर मह धव ा यांनी सुदधा
् राजीनामा दे याची
तरतूद घटनेत नाही.

Question No. 35
Question
69 वी घटना ती 1991 नुसार भारतीय रा यघटनेत खालीलपैक कोणकोण या अनु छे दांचा समावेश कर यात आला आहे ?

(अ)   अनु छे द 239(A)

(ब)     अनु छे द 239(AA)

(क)    अनु छे द 239(AB)

(ड)    अनु छे द 239(B)

Options

a  फ अ आ ण ब
b  फ त अ आ ण क
c  फ ब आ ण क
d  फ अ, ब आ ण क

Your Answer unanswered

Correct Answer c
Explanation
 69 Amendment - 1991 समावेश कलमे 
(1) कलम 239 (AA), (2) कलम 239 (AB)

Question No. 36
Question

पुढ ल वधाने  वचारात या.

(अ)      काही व श प र थतीम ये, रा यसूचीतील कोण याही वषयावर कायदा कर याचा अ धकार भारतीय घटनेने संसदे ला दला
आहे.

(ब)     रा ीय अ◌ाणीबाणी या अंमलबजावणी या काळात संसद रा यसूचीतील कोण याही वषयावर कायदा क शकते.

Options
a  अ आ ण ब दो ही बरोबर आ ण ब हे अ चे यो य प ीकरण आहे.
b  अ आ ण ब दो ही बरोबर परंतु ब हे अ चे यो य प ीकरण नाही.
c  अ बराेबर आहे पण ब चुक चे आहे.
d  अ चुक चे आहे पण ब बरोबर आहे.

Your Answer unanswered

Correct Answer b
Explanation
 

Question No. 37
Question

यो य जो ा जुळवा.

तंभ (I) घटना ती                                                                तंभ (II) भागसमा व   

(अ)   73 वी घटना ती 1992                                                       (i) भाग 9 (A)

(ब)   74 वी घटना ती 1992                                                        (ii) भाग 9 (B)

(क)   42 वी घटना ती 1976                                                        (iii) भाग 9

(ड)    97 वी घटना ती 2011                                                          (iv) भाग 14 (A) 

Options

a  अ-iv, ब-iii, क-ii, ड-i


b  अ-iii, ब-iv, क-i, ड-ii
c  अ-iii, ब-i, क-iv, ड-ii
d  अ-i, ब-ii, क-iii, ड-iv

Your Answer unanswered

Correct Answer c
Explanation
‘‘घटना ती या ही रा यघटनेची स म प रपूण बाजू आहे ती गुंतागुंतीची असली तरी ती एक भ बाब आहे’’ असे
घटना ती ॅ हल ऑ टन यांनी केले आहे. 
येसंबंधीचे वधान न

Question No. 38
Question

 ‘‘घटना ती या ही रा यघटनेची स म प रपूण बाजू आहे ती गं◌ुतागुंतीची वाटत असली तरी ती केवळ एक भ बाब आहे’’
असे वधान घटना तीबाबत खालीलपैक कोणी केलेले अ◌ाहे ?

Options

a  पं. जवाहरलाल नेह


b  के. सी हेअर
c   ॅन हल ऑ टन
d  डॉ. बाबासाहेब आंबेडकर

Your Answer unanswered

Correct Answer c
Explanation
 

Question No. 39
Question

खालील वधाने पहा.

(अ)        अमे रके माणे भारतातही नागरी सेवकां या संपावर तबंध करणारा कोणताही कायदा नाही.

(ब)          शासक य सुधारणा आयोगाने अशी शफारस केली होती क , नागरी सेवका या संपावर भारतात पूणपणे तबंध असावा.

वरीलपैक कोणते/ती वधान/ने बरोबर आहे/ त ?


Options

a  फ अ 
b  फ ब
c  फ अ व ब
d  दो ही नाही.

Your Answer unanswered

Correct Answer b
Explanation
 

Question No. 40
Question

खालीलपैक कोणती जोडी बरोबर नाही?

(अ)     अनु छे द 153 = रा यपालाचे पद

(ब)     अनु छे द 156 = रा यपालाचा कायकाळ

(क)    अनु छे द 154 = रा यपालाचे कायकारी अ धकार

(ड)     अनु छे द 155 = रा यपालाची पद युती

Options

a  फ ब आ ण ड
b  फ अ आ ण क
c  फ ड 
d  एकही नाही.

Your Answer unanswered

Correct Answer c
Explanation
 अनु छे द 154 रा यापालाचे कायकारी अ धकार.
अनु छे द 155 रा यपालाची नेमणूक.
अनु छे द 156 रा यपालाचा कायकाळ.

Question No. 41
Question

 भारता या प र े ा या बाहेर असले या े ावर भारताचे कायदे लागू राहतील अशी तरतूद भारतीय रा यघटनेत कोण या अनु छे दात
आहे ?

Options

a  अनु छे द 259
b  अनु छे द 260
c  अनु छे द 261
d  अनु छे द 361

Your Answer unanswered

Correct Answer b
Explanation
 अनु छे द 260 - भारताबाहेरील े ाबाबत संघरा याचे अ धकार े .
अनु छे द 259 - प र श 1 मधील भाग (B) मधील रा यात स दल ( नर सत केले गेले).
अनु छे द 261 - सावज नक कृती, नोद , आ ण या यक कामकाज.
अनु छे द 361 - रा पती, रा यपाल व रा य मुख यांना संर ण.

Question No. 42
Question

पुढ ल दोन वधानांपैक काेणते चुक चे आहे?

(अ)     सावज नक आरो य व व छता हा संघसूचीतील वषय आहे.

(ब)      कामगार क याण हा वषय संघसूचीम ये नमूद आहे.

 
Options

a  फ अ 
b  फ ब
c  दो ही अ, ब
d  दो ही नाही

Your Answer unanswered

Correct Answer c
Explanation
 सावज नक आराे य व व था हा वषय रा यसूचीतील आहे.
कामगार क याण हा वषय समवत सूचीतील आहे.

Question No. 43
Question

 क रा य संबंधाबाबत खालील वधान काेण या आयोगाने केले ते दले या पयायातून नवडा.


वधान : ‘‘अ तक करणामुळे क ात र दाब वाढतो तर प रघावर र ाचा अभाव नमाण होताे’’

Options

a  पंछ आयोग
b   शासक य सुधारणा अ◌ायाेग
c  राजम ार अ◌ायोग
d  सरकारीया अयोग

Your Answer unanswered

Correct Answer d
Explanation
 ‘‘अ तक करणामुळे क ात र दाब वाढतो तर प रघावर र ाचा अभाव नमाण होतो. - सरका रया अ◌ायोग. 

Question No. 44
Question
यो य जो ा जुळवा.

      तंभ (I)                                                                        तंभ (II)

     प र श                                                                        तरतूद 

(अ)   प र श सातवे                                     (i) कायालयीन भाषा

(ब)     प र श नववे                                        (ii) जमीनसुधारणा कायदा

(क)    प र श बारावे                                     (iii) क आ ण रा यांंम ये अ धकार वाटप.

(ड)     प र श आठवे                                      (iv) नगरपा लकांचे अ धकार अ धस ा आ ण जबाबदारी 

Options

a  अ-iii, ब-ii, क-iv, ड-i


b अ-iii, ब-iv, क-ii, ड-i
 
c  अ-i, ब-ii, क-iii, ड-iv
d  अ-i, ब-iii, क-ii, ड-iv

Your Answer unanswered

Correct Answer a
Explanation
 
परश तरतूद
7 वे क & रा यात अ धकाराचे वाटप
8 वे कायालयीन भाषा
9 वे जमीन सुधारणा कायदा.
 12  वे नगरपा लकांचे अ धकार अ धस ा आ ण जबाबदारी.

Question No. 45
Question
पुढ ल कोणती वधाने यो य आहेत ?

(अ)      1994 म ये थापन झालेली वशेष पोलीस आ थापना स या क य अ वेषण वभागात वलीन झालेली आहे.

(ब)     क य अ वेषण वभाग ही एक वैधा नक सं था आहे.

(क)    द ली वशेष पोलीस आ थापना कायदा 1946 नुसार क य अ वेषण वभागाला अ धकार ा त झालेले आहेत.

Options

a  अ आ ण ब
b  ब आ ण क
c  अ आ ण क
d  अ, ब, क

Your Answer unanswered

Correct Answer c
Explanation
 क य अ वेषण वभाग - 1994 साली थापन झालेली वशेष पोलीस आ थापनेची नवीन आवृ ी
ही वैधा नक सं था नाही.

Question No. 46
Question

नदा ंजक ठराव आ ण अ व ास ठराव यांतील फरकासंदभात खालीलपैक काेणती  वधाने बरोबर आहेत?

(अ)      नदा ंजक ठराव लोकसभा व रा यसभा या दो ही गृहात मांडता येतो, तर अ व ासाचा ठराव फ लोकसभेत मांडता येतो.

(ब)    अ व ासाचा ठराव फ मं मंडळा व द्ध असतो तर नदा ंजक ठराव एका मं या व द्ध कवा मं ीगटा व द्ध ही असू
शकतो. 

(क)    नदा ंजक ठरावाम ये कारण नमूद करणे आव यक आहे,पण अ व ासा या ठरावाम ये कारण नमूद करणे आव यक नाही.

(ड)    अ व ासाचा ठराव संमत झा यास सरकारने राजीनामा दला पा हजे तर नदा जक ठराव संमत झा यास सरकारने राजीनामा
दे णे आव यक नाही.

Options
a  अ, क, ड
b  ब, क, ड
c  अ, ब, ड
d  अ, ब, क

Your Answer unanswered

Correct Answer b
Explanation
 

Question No. 47
Question

 कोण या घटना ती दारे रा ीय आणीबाणी लावताना रा पती या खा ीला कोण याही यायालयात आ हान न दे याची तरतूद
तसेच यांचे या यक पुन वलाेकन न कर याची तरतूद कर यात आली ?

Options

a  38 वी घटना ती 1975


b  42 वी घटना ती 1976
c  44 वी घटना ती 1978
d  24 वी घटना ती 1971

Your Answer unanswered

Correct Answer a
Explanation
 38 Amendment - 1975.
रा पतीने केले या आणीबाणी या घोषणेवर यायालयात जाता येणार नाही अशी तरतूद.
एकाच वेळ नर नरा या कारणासाठ रा ीय आणीबाणी या नर नरा या घोषणा कर याचे अ धकार रा पतीला दे यात
आहे.
रा पती, रा यपाल आ ण संघरा य दे शांचे शासक यांनी जारी केले या वट कुमां व दध यायालयात जाता येणार नाही
अशी तरतूद.

Question No. 48
Question

पुढ लपैक कोणते वधान अयो य आहे ?

(अ)      रा य लोकसेवा आयोगाचे सद य यां या पदावर जू झा यापासून 6 वषाचा कालावधी पूण करेपयत कवा 62 वष वयापयत
पदावर कायरत रा शकतात.

(ब)     जर एखादा सद य नवृ ीनंतर महारा लोकसेवा आयोगाचा अ य हणून नयु झाला तर याचे वय कतीही असले तरी
याला अ धकचा सहा वषाचा कायकाळ मळतो.

Options

a  फ अ
b  फ ब
c  अ व ब दो ही
d  दो ही नाहीत.

Your Answer unanswered

Correct Answer b
Explanation
 जर एखाद रा य लोकसेवा आयोगाचा अ य हणून नयु झालेली चे वय 62 वषा या आत असेल तोपयतच
पदावर रा शकते. तला तसा अ धकचा 6 वषाचा कायकाळ मळत नाही.

Question No. 49
Question
यो य जो ा जुळवा.

                          तंभ (I)                                                                                  तंभ (II)

                    वषय                                                                                         स मती

(अ)     लाेकशाही आ ण वकासासाठ पंचायती                     (i) जी. ही.के राव

        राज सं थांत नवचैत य आणणे.

(ब)     सामुदा यक वकास काय म आ ण                             (ii) बलवंतराय मेहता

         रा ीय व तार सेवा.

(क)    पंचायतीराज सं था                                                          (iii) अशोक मेहता

(ड)      ामीण वकास आ ण दा र य नमूलनां या               (iv) एल.एम. सघवी

       स या या शास नक व था 

Options

a  अ-iv, ब-ii, क-iii, ड-i


b  अ-iv, ब-iii, क-ii, ड-i
c  अ-iii, ब-ii, क-iv, ड-i
d  अ-iii, ब-iv, क-ii, ड-i

Your Answer unanswered

Correct Answer a
Explanation
 

Question No. 50
Question
 अनु छे द 28 नुसार थापन कर यात आले या पुढ लपैक काेण या शै णक सं थेम ये श ण दे ता येत नाही ? 

(अ)    पूणपणे रा यसं थे या कवा सरकारी पैशातून चाल व या जाणा या शै णक सं था.

(ब)      रा यसं थेमाफत शा सत मा दान नधी कवा यास हणून थापन केले या सं था.

(क)    रा यसं थेची मा यता मळ वणा या सं था.

(ड)     रा यसंसथेचे साहा य मळ वले या सं था.

Options

a  फ अ 
b  फ अ, क, ड
c  फ अ, ब, क
d  वरीलपैक सव

Your Answer unanswered

Correct Answer a
Explanation
 कलम 28 नुसार थापन कर यात आले या खालील शै णक सं थेत धा मक श ण दे ता येते.
(1) रा यसं थेमाफत शा सत मा अनुदान कवा यास हणून थापन कर यात आले या सं था.
(2) रा यसं थेची मा यता मळ वणा या सं था.
(3) रा यसं थेचे साहा य मळ वले या सं था.

Question No. 51
Question
 रा पतीने आ थक आणीबाणी घो षत के यास याचे खालीलपैक काय प रणाम होतील ? 

(अ)      आ थक वहारातील सु पणा या त वांचे पालन कर यासंबंधी रा पती रा यांना आदे श दे ऊ शकतील.

(ब)        सव च यायालय आ ण उ च यायालयांचे यायाधीश सोडू न रा पती सरकारी कमचा यां या वेतन आ ण भ यात कपात क
शकतात.

(क)      रा य व धमंडळानी संमत केलेली सव अथ वधेयके आ ण इतर व ीय वधेयके रा पती या  वचाराथ राखून ठे वली जाऊ
शकतात.

(ड)    भारता या सं चत नधीतून खचाला मा यता दे याचा अ धकार संसद रा पतीला दे ऊ शकते.

Options

a  फ अ, ब, क
b  अ, ब, क, ड
c  फ अ आ ण क
d  फ अ,क, ड

Your Answer unanswered

Correct Answer c
Explanation
रा पतीने आ थक आणीबाणी घो षत के यास.
(1) आ थक वहारातील सु पणा या त वांचे पालन कर यासंबंधी रा पती रा यांना आदे श दे ऊ शकतात.(2) भारता या सं चत
नधीतून खचाला मा यता दे याचा अ धकार संसद रा पतीला दे ऊ शकत नाही.
 

Question No. 52
Question
यो य जो ा जुळवा.

                  तंभ (I)                                                                                                                      तंभ (II)         

                  तरतूद                                                                                                                      अनु छे द

(अ)    रा ीय हतासाठ रा यसूचीतील                                                                        (i) अनु छे द 248

बाबीसंबंधी व ध नयम करणे.

(ब)    समवत सूची कवा रा यसूचीमधे                                                                       (ii) अनु छे द 249

           नमूद न केले या कोण याही

      बाबीसंबंधी  व ध नयम करणे.

(क)   आंतररा ीय करारांची अंमलबजावणी                                                             (iii) अनु छे द 250

         कर यासाठ व ध नयम करणे.

(ड)    आणीबाणीची घोषणा जारी असतांना                                                                 (iv) अनु छे द 253

रा यसूचीतील कोण याही बाबीसंबंधी  व ध नयम करणे. 

Options

a  अ-i, ब-ii, क-iii, ड-iv


b  अ-ii, ब-i, क-iv, ड-iii
c  अ-iii, ब-ii, क-iv, ड-i
d  अ-iv, ब-iii, क-ii, ड-i

Your Answer unanswered

Correct Answer b
Explanation
 

Question No. 53
Question
मूलभूत अ धकारातील अनु छे द 22 म ये खालीलपैक कोणती तरतूद समा व नाही ?

(अ)     काेण याही ला अटक करतेवेळ अटक कर याची कारणे प केली जातील ; तसेच या ला ह ा असले या
व कलामाफत वत:ची बाजू मांड याचे वातं य दले जाईल. 

(ब)    अटक केले या येक स अटक के यापासून 24 तासां या आत सवात नजीक या यायाधीशांसमोर हजर केले जाईल.

(क)    एखादया गु हयाबद्दल आरोपी असले या ला वत: व द सा दे याची स केली जाणार नाही.

Options

a  अ आ ण क
b  फ ब 
c  फ क
d  एकही नाही.

Your Answer unanswered

Correct Answer c
Explanation
 एखा ा गु ाबददल आरोपी असले या ला वत: व द्ध सा दे याची स केली जाणार नाही अशी तरतूद
अनु छे द 22 मधे केलेली नाही.

Question No. 54
Question

खालील वधाने वचारात या.

(अ)    रा पती आ ण रा यपाल यांचा अ यादे श काढ याचा अ धकार समान आहे.

(ब)     रा पती आपला अ यादे श मागे घेऊ शकतो परंतु रा यपाल एकदा काढलेला अ यादे श मागे घेऊ शकत नाही.

Options

a   वधान अ बराेबर ब चुक चे.


b   वधान अ चुक चे ब बरोबर 
c  दो ही वधाने चुक ची 
d  दो ही वधाने बरोबर

Your Answer unanswered

Correct Answer c
Explanation
 रा पती आ ण रा यपाल यांचा अ यादे श काढ याचा अ धकार समान नसून वेगळा आहे.
एखादा काढलेला अ यादे श रा यपाल मागे घेऊ शकतात. पण असा अ यादे श रा पती मागे घेऊ शकत नाही. 

Question No. 55
Question

 ‘‘अ यादे श काढ याची रा पतीची खा ी अयो य हेतू अस याकारणाव न या अ यादे श काढ या या खा ीला यायालयात आ हान
दे ता येऊ शकते’’ असा नणय सव च यायालयाने खालीलपैक कोण या खट यात दला ?

Options

a  ए.के. गोपालन खटला -1950


b  गोलकनाथ खटला - 1967
c  डी. सी. वधवा खटला - 1987
d  कुपर खटला - 1970

Your Answer unanswered

Correct Answer d
Explanation
 ‘‘अ यादे श काढ या या रा पतीची खा ी मागे अयो य हेतू अस यास या अ यादे श काढ या या खा ीला यायालयात
आ हान दे ता येऊ शकते’’ असे सव च यायालयाने 1970 या कूपर खट यात प केले.

Question No. 56
Question

 संयु लोकसेवा आयोगा या अ य ांचा कायकाळ कती असतो ते दले या पयायातून नवडा.

Options
a  5 वष कवा वयाची 65 वष 
b  5 वष कवा वयाची 62 वष 
c  6 वष कवा वयाची 65 वष 
d  6 वष कवा वयाची 62 वष 

Your Answer unanswered

Correct Answer d
Explanation
 क य लाेकसेवा आयोगा या अ य ांचा कायकाळ - 6/65 वष.
रा य लोकसेवा आयोगा या अ य ं◌ाचा कायकाळ - 6/62 वष.
संयु रा य लाेकसेवा आयोगा या अ य ांचा कायकाळ हा रा य लोकसेवा आयोगा या अ य ांएवढाच असतो. हणून उ र
- 6/62 वष (जे अगोदर असेल ते).

Question No. 57
Question

यो य जो ा जुळवा.

        तंभ (I)                                                                                                                              तंभ (II)

   संसद य स म या                                                                                                                थापना 

(अ) सावज नक लेखा स मती.                                                                                         (i) 1964

(ब) सावज नक उप म स मती.                                                                                       (ii) 1953

(क) यम काय ासंबंधी स मती.                                                                               (iii) 1921

(ड) सरकारी आ ासनांसंबंधी स मती.                                                                         (iv) 1953

Options

a  अ-iii, ब-i, क-iv, ड-ii


b  अ-iii, ब-ii, क-i, ड-iv
c  अ-ii, ब-iii, क-i, ड-iv
d  अ-iii, ब-ii, क-iv, ड-i

Your Answer unanswered


Correct Answer a
Explanation
 सावज नक लेखा स मती = थापना 1921
सावज नक उप म स मती = थापना 1964
यम कायद्या संबंधी स मती = थापना 1953
सरकारी आ ासनासंबंधी स मती = थापना 1953

Question No. 58
Question

खालील वा ये वाचा आ ण दले यांपैक यो य पयाय नवडा :

(अ)    24 जानेवारी 1950 रोजी घटनास मतीने रा गान वीकृत केले.

(ब)    24 जानेवारी 1950 रोजी घटनास मतीने रा गीत वीकृत केले.

(क)    22 जुलै 1947 रोजी घटना स मतीने रा ीय वज वीकृत केला.

(ड)    24 जानेवारी 1950 रोजी घटना स मतीने डॉ.राज साद यांना भारताचे प हले रा पती हणून नवडू न दले.

Options

a   वधान क हे अस य आहे.
b   वधाने ब आ ण क ही अस य आहेत.
c   वधान ड हे अस य आहे.
d  सव वधाने स य आहेत.

Your Answer unanswered

Correct Answer d
Explanation
 24 जानेवारी 1950 - रा गान वीकृत.
24 जानेवारी 1950 - रा गीत वीकृत.
24 जुलै 1947 - रा ीय वज वीकृत
24 जानवारी 1950 भारताचे प हले रा पती हणून डॉ. राज साद नवडू न आले.
Question No. 59
Question

संसदे संदभात खालीलपैक कोणते/ती वधान/ने स य आहे./त/ ?

(अ)     17 ए ल 1952 पासून हंगामी संसदे चे अ त व संपु ात आले.

(ब)      दोन सदने असलेली प हली नवडू न आलेली संसद मे 1954 म ये अ त वात आली.

Options

a   वधान अ
b   वधान ब
c वधान अ आ ण ब  
d  एकही वधान नाही

Your Answer unanswered

Correct Answer a
Explanation
 दोन सदने असलेली प हली नवडू न आलेली संसद मे 1952 मधे आ त वात आली.
17 ए ल 1952 पासून हंंगामी संसदे चे आ त व संपु ात आले.

Question No. 60
Question
यो य जो ा जुळवा.

                        तंभ (I)                                                                          तंभ (II)

                    स मती                                                                              अ य

(अ)    कामकाज म स मती.                                             (i) एस. वारदाचारी

(ब)     रा वजसंबंधी हंगामी स मती.                                (ii) ग. वा. मावळणकर

(क)    घटनास मती या काया वषयी स मती.                    (iii) डॉ. राज साद

(ड)   सव च यायालयाबाबत हंगामी स मती.                    (iv) डॉ. के. एम. मुंशी

Options

a  अ-iv, ब-iii, क-ii, ड-i


b  अ-i, ब-ii, क-iii, ड-iv
c  अ-iv, ब-ii, क-iii, ड-i
d  अ-i, ब-iii, क-ii, ड-iv

Your Answer unanswered

Correct Answer a
Explanation
संघरा यीय अ धकार स मती = पं डत जवाहरलाल नेह .
ां तक रा यघटना स मती = सरदार पटे ल.
मसुदा स मती = डॉ. बाबासाहेब आंबेडकर.
कामकाज या नयमन स मती = डॉ. राज साद.
वृ प क स मती = उषानाथ सेन.
सुकाणू स मती = डॉ. राज साद.
 

Question No. 61
Question

 खालीलपैक कोण या लेखकाने या या ‘‘ सपल ऑफ सोशल अँड पॉ ल टक़ल थेअरी’’ या पु तका या सु वातीला भारतीय
उददे शप का उद्धृत केली आहे ? 

Options
a  अन ट बाकर
b  डी.डी.बासू
c  अम य सेन
d   ॅन हील ऑ टन

Your Answer unanswered

Correct Answer a
Explanation
 अन ट बाकरने या या ‘‘ सपल ऑफ सोशल अँड पॉ ल टकल थेअरी’’या पु तका या सु वातीला भारतीय उद्दे शप का
उद्धृत केलेली आहे.

Question No. 62
Question

खालील वधाने ल ात या.

(अ)    रा यघटने या व वध भागात केले या अनेक व मह वा या बदलांमुळे 42 ा Amendment ला ‘‘लघु रा यघटना’’ हटले जाते.

(ब)    ऑ े लयन रा यघटनेत सु वातीला 128 तर चीन या घटनेत 138 कलमे होती.

वरील वधानांतील चुक चे वधान/ने कोणती ?

Options

a  अ आ ण ब दो ही नाही
b  फ अ
c  फ ब 
d  अ आ ण ब दो ही

Your Answer unanswered

Correct Answer a
Explanation
 42 वी Amendment - 1976
हलाच Mini Constitution हटले जाते.
या Amendment दारे वण सग स मती या शफारशी लागू कर यात आ या.
हनेच घटने या सरना यात समाजवाद ,धम नरपे आ ण एका मता हे तीन नवीन श द समा व केले.
मं मंडळाचा स ला रा पतीवर बंधनकारक कर यात आला.
सव च आ ण उ च यायालयाचे या यक पु्न वलाेकनाचे आ ण ा धलेख काढ याचे अ धकार े मया दत कर यात आले.
रा यातील रा पती राजवट चा कालावधी एकावेळ सहा म ह याऐवजी एकावेळ एक वषाने वाढ वता येईल अशी तरतूद
कर यात आली.
अ खल भारतीय या यक सेवा नमाण कर यात आली. अशा कार या अनेक मह वा या तरतुद या Amendment दारे
कर यात आ यानेच तला ‘‘लघु रा यघटना’’ हणतात.

Question No. 63
Question

भारतीय रा यघटना ही जगातील एक अवाढ रा यघटना आहे ; या अवाढ आकार अस याची खालीलपैक कोणती कारणे आहेत
?

(अ)    दे शाचा चंड आकार व यातील ोद शक वै व य यासारखे भौगो लक घटक.

(ब)    घटनास मतीत असलेले कायदे त ांचे वच व.

(क)    क व रा यांसाठ एकच रा यघटना.

(ड)   1935 या भारत सरकार काय ाचा आप या घटनेवरील भाव व इतर ऐ तहा सक घटक.

Options

a  फ अ आ ण क 
b  फ अ आ ण ड
c  फ अ, क, ड
d  वरीलपैक सव

Your Answer unanswered

Correct Answer d
Explanation
 भारतीय घटना अवाढ अस याची कारणे:
(1) 1935 या काय ाचा भारतीय घटनेवरील भाव.
(2) दे शाचा चंड आकार व ादे क वै व य.
(3) घटनास मतीत असलेले कायदे त ं◌ाचे वच व.
(4) क व रा यासाठ एकच रा यघटना.
Question No. 64
Question

खाली दले या वधानांपैक कोणती वधान/ने बनचूक नाहीत ?

(अ)    एखा ा रा याची भौगो लक सलगता व याचे अ त व याची भारतीय रा यघटना  हमी दे त नाही.

(ब)     भारतीय संघरा याचे ‘‘बदलणा या घटकरा यांचा एक अ वचल संघ’’ असे वणन यो य ठरते.

Options

a  फ अ
b  फ ब
c  अ व ब दो ही
d  अ व ब दो ही नाही.

Your Answer unanswered

Correct Answer d
Explanation
एखा ा रा याची भौगो लक सलगता व या रा याचे अ त व याची भारतीय घटना हमी दे त नाही कारण या रा यापासून
वेगळे रा य नमाण कर याचा अ धकार संसदे ला आहे; हणून जर एखादय्◌ा रा यापासून सरे नवीन रा य नमाण केले तर
रा याची भौगो लक सलगता (नकाशा) बदलते. के. हैनुमंत या यांनी केले आहे.
 

Question No. 65
Question
यो य जो ा जुळवा.

                  तंभ (I)                                                                                                                        तंभ (II)

                    तरतूद                                                                                                                           अनु छे द

(अ)       रा पती या शफारशी शवाय                                                                                  (i) अनु छे द 117

               खचासाठ काेणतीही मागणी केली जाणार नाही.

(ब)          व धमंडळा या मा यते शवाय                                                                                 (ii) अनु छे द 265

           कोणताही खच केला जाणार नाही.

(क)       कोणताही कर कायदे शीर अ धकारा शवाय                                                              (iii) अनु छे द 266

             आकारला जाणार नाही व संक लत केला  जाणर नाही.

(ड)        रा पती या शफारशी शवाय कर                                                                              (iv) अनु छे द 113                               


                                                                                           आकार यासंबंधी कोणतेही वधेयक

              संसदे त  सादर केले जाणार नाही.

Options

a  अ-iv, ब-iii, क-ii, ड-i


b  अ-i, ब-ii, क-iii, ड-iv
c  अ-iv, ब-ii, क-iii, ड-i
d  अ-i, ब-iii, क-ii, ड-iv

Your Answer unanswered

Correct Answer a
Explanation
 

Question No. 66
Question
भारतीय रा यघटनेसंदभात खालील वधान कोणी केले ?

वधान : ‘‘आ हाला वीणा कवा सतार यांचे संगीत हवे होते, पण येथे इं जी घोष वभागाचे संगीत आहे. घटनाकारांचे श णच या
प दतीने झालेले आहे’’

Options

a  लोकनाथ म
b  ल मीनारायण सा
c  के. बी. कामत
d  के. हनुमंत या

Your Answer unanswered

Correct Answer d
Explanation
 ‘आ हाला वीणा कवा सतार यांचे संगीत हवे होते पण येथे इं जी घोष वभागाचे संगीत आहे’ घटनाकारांचे श णच या
पदधतीने झालेले आहे’’ भारतीय रा यघटनेचे वणन ‘बदलणा या घटकारा याचा अ वचल संघ’ असे यो य ठरते.

Question No. 67
Question

खालीलपैक कोणते श द फ उददे शप केत नमूद असून ते इतर नाहीत ?

(अ)    सावभौम

(ब)    समाजवाद

(क)    धम नरपे

(ड)   लाेकशाही. 

Options

a  फ अ, ब, क
b  फ बआणक
c  फ ब, क, ड
d  वरीलपैक सव

Your Answer unanswered

Correct Answer b
Explanation
 समाजवाद आ ण धम नरपे हे दोन श द फ उदे दशप केत नमूद आहेत, इतर नाहीत.

Question No. 68
Question

 खालील वधानांची स यता तपासा. खालीलपैक काेणते वधान बरोबर आहे ते आ◌ेळखा.

वधान (अ) 1973 साली लॅकद व, मनीकॉय, अ मनद वी या बेटांचे ‘ल दप’ असे नामकरण कर यात आले.
वधान (ब) 1973 साली मैसूरचे नामकरण कनाटक कर यात आले.

Options

a   वधान अ आ ण ब दो ही बरोबर आहेत.


b   वधान अ आ ण ब दो ही चूक आहेत.
c   वधान अ बरोबर असून वधान ब चूक आहे.
d   वधान अ चूक असून वधान ब बरोबर आहे.

Your Answer unanswered

Correct Answer a
Explanation
 1973 - लॅेकद व म नकॉय, अ मनद वी या बेटांचे ल द प असे नामकरण.
1973 - यावष हैसूर शहराचे नामकरण कनाटक असे कर यात आले.

Question No. 69
Question
रा यांची पुनरचना करताना भाषांचा  आधार का घे यात आला ?

(अ)    यामुळे टशांनी ल त केले या था नक भाषा सुधारतील अशी अपे ा होती.

(ब)     भा षक आधारामुळे था नक जनतेचा था नक शासनात जा तीचा सहभाग करता येणार होता.

(क)    भा षक दे श नैस गक तसेच भौगो लक या सलग होते.

(ड)    1947 पूव पासूनच ातांची रचना भाषे या आधारावार करा अशी मागणी होती.

Options

a  फ अ, ब, क
b  फ ब, क, ड
c  फ अ, क, ड
d  वरील सव.

Your Answer unanswered

Correct Answer d
Explanation
 रा याची पुनरचना करताना भाषांचा आधार घे यात आला.
(1) टशानी ल त केले या था नक भाषा सुधारतील अशी अपे ा होती.
(2) भा षक आधारामुळे था नक जनतेचा था नक शासनात जा तीचा सहभाग करता येणार होता.
(3) भा षक दे श नैस गक तसेच भौगो लक या सलग होते.

Question No. 70
Question
यो य जो ा जुळवा.

                    तंभ (I)                                            तंभ (II)

                  भाग                                                   समावेश

(अ) भाग अ मधील रा ये                       (i) राज मुखांचे रा य

(ब) भाग ब मधील रा ये                           (ii) ग हनसचे रा य

(क) भाग क मधील रा ये                      (iii) क मशनरांचे रा य

(ड) भाग ड मधील रा ये                         (iv) अंदमान व नकोबार

Options

a  अ-ii, ब-i, क-iii, ड-iv


b  अ-i, ब-ii, क-iii, ड-iv
c  अ-iii, ब-ii, क-i, ड-iv
d  अ-ii, ब-iii, क-i, ड-iv

Your Answer unanswered

Correct Answer a
Explanation
 भाग अ रा ये = ग हनसचे रा य
भाग ब रा ये = राज मुखाचे रा य
भाग क रा ये = क मशनरांचे रा य
भाग ड रा ये = अंदमान नकोबार.

Question No. 71
Question

 रा पती व रा यपाल यां या मादाना या अ धकारासंबं धत खालील वधाने वचारात या.

वधान (अ) : रा पत ना ल करी यायालयाने दले या श ेबाबत कवा श ादे शाबाबत मादान कर याचा अ धकार आहे.

वधान (ब) : रा यपालास मृ युदंडाचा श ादे श नलं बत कर याचा, यात सूट दे याचा कवा तो सौ य कर याचा अ धकार आहे.
Options

a   वधान अ आ ण ब दो ही बरोबर आहेत.


b   वधान अ आ ण ब दो ही चूक आहेत.
c   वधान अ बरोबर असून वधान ब चूक आहेत.
d   वधान अ चूक असून वधान ब बरोबर आहे.

Your Answer unanswered

Correct Answer a
Explanation
रा यपालचा मादानाचा अ धकार अनु छे द 161 म ये स व तर दे यात आला आहे.
रा ापत चा मादानाचा अ धकार अनु छे द 72 मधे स व तर दे यात आला आहे. 

Question No. 72
Question

 भारतीय रा यघटने या अनु छे द 123 संदभात खालीलपैक कोणते वधान बनचूक नाही ? 

(अ)   रा पत ना अ यादे श केवळ संसदे या वरामकाळातच काढता येतो.

(ब)    रा पती अ यादे श अशाचवेळ काढू शकतात, जे हा यांनी ता काळ कायवाही करणे गरजेचे आहे.

(क)    रा पतीवर महा भयोग लाव यासाठ घटनाभंगाचा दोषारोपाचा ठराव आधी लोकसभेत मांडावा लागतो.

(ड)    उपरा पतीस यां या पदाव न र कर यासाठ या आशयाचा ठराव थम रा यसभेत पा रत करावा लागतो.

Options

a  फ अ आ ण क
b फ अ आ ण ड 
c  फ क
d  वरीलपैक सव

Your Answer unanswered

Correct Answer c
Explanation
 रा पतीवरील महा भयोगाचे आरोपप संसदे या कोण याही गृहात मांडले जाऊ शकते.
या गृहात असे आरोपप मांडले जाईल या गृहातील 1/4 सद यांनी आरोपप ावर वा री केली पा हजे आ ण रा पतीला
14 दवसांची पूवसूचना दली पा हजे.
महा भयोग ही संसदे तील अध या यक या आहे.

Question No. 73
Question

यो य जो ा जुळवा.

            तंभ (I)                                                  तंभ (II)

           अनु छे द                                                     तरतूद

(अ)    अनु छे द 22                         (i) जी वत व गत वातं य यांचे संर ण.

(ब)    अनु छे द 23                         (ii) अपराधा या दोषा स द बाबत संर ण.

(क)    अनु छे द 20                        (iii) माणसांचा अप ापार व वेठ बगारी यांना मनाई.

(ड)   अनु छे द 21                           (iv) वव त करणी अटक व थानबद्धता यांपासून संर ण

Options

a  अ-iv, ब-iii, क-ii, ड-i


b  अ-i, ब-ii, क-iii, ड-iv
c  अ-iv, ब-ii, क-iii, ड-i
d  अ-i, ब-iii, क-ii, ड-iv

Your Answer unanswered

Correct Answer a
Explanation
 वा षक व ीय ववरणावर रा यसभेत चचा होते ; मा मतदान होत नाही. 

Question No. 74
Question
खालील वधाने वचारात या.

(अ)    धन वधेयकात ती सुचव याचे वा ते फेटाळ याचे अ धकार रा यसभेला नाहीत.

(ब)     वा षक व ीय ववरणावर रा यसभेत चचा आ ण मतदान होत नाही.

(क)    मं मंडळावरील अ व ासाचा ठराव संमत कर याचा अ धकार रा यसभेला नाही. 

वरीलपैक काेणते/ती वधान/ने चुक ची/चे आहे/त ?

Options

a  फ अ
b  फ ब
c  फ क
d  वरीलपैक काेणतेही नाही.

Your Answer unanswered

Correct Answer b
Explanation
 वा षक व ीय ववरणावर रा यसभेत चचा होते ; मा मतदान होत नाही.

Question No. 75
Question

 खालीलपैक काेण या संसद य स मतीम ये फ लाेकसभेतीलच सद य असतात ?

Options

a  लाेकलेखा स मती
b  लाेकउप म स मती
c  अंदाज स मती
d  वरीलपैक सव

Your Answer unanswered


Correct Answer c
Explanation
अंदाज स मती -
फ लाेकसभेतीलच सद यांचा समावेश.
रा यसभा आ ण लोकसभा अशा दो ही सभागृहातील सद यांचा समावेश असले या स म या-
(1) लोकलेखा स मती, (2) लोकउप म स मती.
 

Question No. 76
Question

 अयो य जोडी ओळखा.

Options

a  अनु छे द 169 = वधानप रषदांची रचना.


b  अनु छे द 170 = वधानसभांची रचना.
c  अनु छे द 165 = रा याचा महा धव ा.
d  अनु छे द 168 = रा यांमधील वधानमंडळाची रचना.

Your Answer unanswered

Correct Answer a
Explanation
 अनु छे द 169 - रा यामधील वधानप रषद वस जत करणे कवा नमाण करणे.

Question No. 77
Question
खालील वधाने वचारात या.

(अ)    ज हा प रषद अ य हाच थायी स मतीचा अ य असतो.

(ब)      ज हा प रषदे ची रचना झा यानंतर ज हा धकारी हे अ य व उपा य यां या नवडणुक साठ सभा बाेलावतात.

(क)    2011 पासून महारा ातील था नक सं था या नवडणुक त म हलांसाठ 50% आर ण लागू केले आहे.

वरीलपैक बनचूक वधान/ने नवडा.

Options

a फ अआणब
b फ अआणक
c फ बआणक
d वरीलपैक सव

Your Answer unanswered

Correct Answer d
Explanation
थायी स मतीचा अ य हा ज हा प रषदे चा अ य असतो.
2011 पासून महारा ातील था नक सं था या नवडणुक त म हलांसाठ 50%आर ण लागू केला.

Question No. 78
Question

खालीलपैक कोणती/ते वधान/ने स य आहेत ?

(अ)    रा य शासक य याया धकरणा या अ य ांची नयु रा यपाल करतो.

(ब)    रा य शासक य याया धकरणां या अ य ाला फ रा पतीच पदाव न र क शकताे.

Options
a अ आ ण ब बराेबर
b अ बरोबर आ ण ब चूक
c अ चूक आ ण ब  बराेबर
d अ आ ण ब चूक

Your Answer unanswered

Correct Answer c
Explanation
रा य शासक य याया धकरणा या अ य ांची नेमणूक/ नयु रा पती करतात.

Question No. 79
Question

खालीलपैक महानगरपा लकेचे अ नवाय नसलेले काम कोणते ?

(अ)   शहरांची सीमा दशवणारी भ कम सीमा च ह उभारणे.

(ब)    ाथ मक श णासाठ शाळा चालवणे.

(क)   सावज नक प रवहन व था उभारणे.

(ड)   ज म व मृ यूची नोद ठे वणे.

Options

a  फ अ 
b  फ ब
c  फ क
d  कोणतेही नाही.

Your Answer unanswered

Correct Answer c
Explanation
 सावज नक प रवहन व था उभारणे हे महानगरपा लकेचे अ नवाय काय नसून ते ऐ छक काय आहे.

Question No. 80
Question

 खालीलपैक काेण या वष ‘‘आंतररा यीय प रषदे ची’’ थापना कर यात आली ?

Options

a  1990
b  1980
c  1992
d  1999

Your Answer unanswered

Correct Answer a
Explanation
आंतररा यीय प रषदे ची थापना = 1990
आंतररा ीय प रषदे संबंधी भारतीय रा यघटनेत अनु छे द 263 मधे स व तर मा हती सां गतलेली आहे.

Question No. 81
Question

 खाली दलेले वधान (A) व कारण (R) वाचा व यो य पयायाची नवड करा.

वधान (A) संसद य णालीम ये व धमंडळ आ ण कायकारी वभागाम ये सलो याचे संबंध व सहकाय असेल.
कारण (R) कायकारी वभाग हा व धमंडळाचा भाग असतो.
Options

a   वधान A आ ण कारण R दो ही यो य असून R हे A चे सुयो य कारण आहे.


b   वधान A आ ण कारण R दो ही यो य असून R हे A चे सुयो य कारण नाही.
c वधान A बरोबर ; परंतु कारण R चूक आहे. 
d  यापैक नाही.

Your Answer unanswered

Correct Answer a
Explanation
 

Question No. 82
Question

खालील वधाने वचारात या.

(अ)    रा यपालाची नयु या बाबतीत भारताने कॅनडाची प दत वीकारली आहे.

(ब)     रा यापालाची नयु रा पती दारे घटनेने ठरवून दले या कालावधीसाठ केली जाते.

(क)    महालेखापालाची नयु करताना रा पती रा यांशी वचार व नमय क शकतात ;  परंतु रा यांचा स ला यां यावर बंधकारक
नाही.

वरीलपैक काेणते वधान/ने चुक चे आहे ?

Options

a  फ अ
b  ब आ ण क
c  फ क
d  यापैक नाही

Your Answer unanswered

Correct Answer b
Explanation
 रा यपालांचा कालावधी घटनेत नमूद नाही.

Question No. 83
Question

खालीलपैक काेणते/ती वधान/ने स य आहेत ?

(अ)   अ व ासाचा ठराव रा यसभा वीकारत नाही.

(ब)     अनु छे द 75 नुसार सरकार हे य नवडू न आले या सद यां या सभागृहालाच सामाईकपणे जबाबदार असते.

Options

a  अ आ ण ब बराेबर 
b  अ बरोबर आ ण ब चूक
c  अ चूक आ ण ब बराेबर
d अ आ ण ब चूक
 

Your Answer unanswered

Correct Answer a
Explanation
 

Question No. 84
Question
यो य जो ा जुळवा.

    तंभ (I)                                                                                तंभ (II)

        वष                                                                                             रा य न मती

(अ) 1963                                                                (i) हमाचल दे श (क. दे श) रा याचा दजा.

(ब) 1966                                                                  (ii) मझोराम रा य नमाण.

(क) 1970                                                                 (iii) नागालँड वतं रा य नमाण.

(ड) 1986                                                                   (iv) पंजाब, ह रयाणा रा य नमाण

Options

a  अ-i, ब-ii, क-iii, ड-iv


b  अ-i, ब-iii, क-ii, ड-iv
c  अ-iii, ब-iv, क-i, ड-ii
d  अ-iii, ब-i, क-iv, ड-ii

Your Answer unanswered

Correct Answer c
Explanation
 1963 - नागालड वतं रा य नमाण.
1966 - पंजाब, ह रयाणा रा य नमाण
1970 - हमाचल दे शाला क. दे श दजा
1980 - मझोराम रा य नमाण.

Question No. 85
Question

 खालील वधानांची स यता तपासा खालीलपैक काेणते वधान बरोबर आहे ते आ◌ेळखा.

वधान (अ) : भारतीय संघरा याचे वणन ‘ वनाशी रा यांचा अ वनाशी संघ’ असे केले जाते.
वधान (ब) : अमे रकन संघरा याचे वणन ‘अ वनाशी रा याचा अ वनाशी संघ’ असे केले जाते.
Options

a   वधान अ आ ण ब दो ही बरोबर आहेत.


b   वधान अ आ ण ब दो ही चूक आहेत.
c   वधान अ बराेबर असून वधान ब चूक आहे.
d   वधान अ चूक असून वधान ब बराेबर आहे.

Your Answer unanswered

Correct Answer a
Explanation
 

Question No. 86
Question

यो य जो ा जुळवा.

      तंभ (I)                                                                          तंभ (II)

      पंचायतराज                                                                          रा य

(अ) मंडल पंचायत                                                          (i) आं दे श

(ब) तालुका पंचायत                                                        (ii) कनाटक

(क) आंच लक पंचायत                                                   (iii) गुजरात

(ड) मंडल जा प रषद                                                      (iv) आसाम

Options

a  अ-iv, ब-i, क-ii, ड-iii


b  अ-ii, ब-iv, क-iii, ड-i
c  अ-iv, ब-iii, क-ii, ड-i
d  अ-ii, ब-iii, क-iv, ड-i

Your Answer unanswered

Correct Answer d
Explanation
 

Question No. 87
Question

ाम यायालयासंदभात खालीलपैक कोणते वधान बनचूक नाही ?

(अ)      ाम यायालयांना फ दवाणी यायालयाचे अ धकार असतात.

(ब)      रा य सरकार उ च यायालयाशी वचार व नमय क न ाम यायालया या पीठासीन अ धका याची नयु करते.

(क)    ाम यायालयाचे मु यालय म यम तरावर या पंचायती या ठकाणी असेल.

Options

a  फ अ
b  फ अ आ ण ड
c  फ अ, क, ड
d  वरीलपैक सव

Your Answer unanswered

Correct Answer a
Explanation
 

Question No. 88
Question
घटनास मतीबाबतची खालील वधाने वाचा आ ण दले यापैक यो य पयाय नवडा.

(अ)      एच. ही.आर.अ यंगार हे घटनास मतीचे स चव होते.

(ब)     घटनास मतीत एस.एन.मुखज हे मसुदा बन व याचे मुख होते.

(क)     मूळ भारतीय रा यघटना ‘ ेम बहारी नारायण रायजादा’ यां या ह ता रात अ◌ाहे.

Options

a   वधान क अस य आहे.
b   वधान ब आ ण क अस य आहेत
c   वधान अ अस य आहे.
d सव वधाने स य आहेत.
 

Your Answer unanswered

Correct Answer d
Explanation
 

Question No. 89
Question

 भारतीय रा यघटनेसंदभात ‘ट के या व पात’ खालील वधान कुणाचे आहे ?

वधान : ‘‘प मेचे गुलामी अनुकरण या न प मेला गुलामी शरणागती’’

Options

a ल मीनारायण सा  
b  लाेकनाथ म  
c  लाॅड वसकाऊंट
d  पी. आर. दे शमुख,

Your Answer unanswered


Correct Answer b
Explanation
 ‘‘प मेचे गुलामी अनुकरण या न प मेला गुलामी शरणागती’ अशी ट का भारतीय रा यघटनेवर लोकनाथ म यांनी
केली आहे.

Question No. 90
Question

पुढ लपैक काेणती वधाने यो य आहेत ?

(अ)    भारतीय घटना लवचीक आहे पण ताठर नाही.

(ब)     भारतातील याय व था वतं आहे पण एका मक नाही.

(क)    भारतीय घटना ताठर आहे पण लवचीक नाही.

(ड)    भारतातील याय व था वतं आहे व एका मक सुदधा


् आहे.

Options

a  अ आ ण क
b  क आ ण ड
c  फ ड
d  वरीलपैक सव

Your Answer unanswered

Correct Answer c
Explanation
 भारतीय घटना लवचीक तसेच ताठरही आहे.
भारतीय याय व था वतं आहे व एका मकही आहे.

Question No. 91
Question
 पुढ लपैक काेणती वधाने बराेबर आहेत ? 

(अ)     घटनास मतीचे स चव हणून ‘अनंतशाम अ यंगार’ यांनी काय केले.

(ब)      घटनास मतीचे उपस चव हणून ‘जुगल कशोर ख ा’ यांनी काय केले.

(क)     घटनास मती या प ह या बैठक त पारशी समाजांचे त न ध व एम.पी.मोद यांनी केले.

(ड)      घटनास मती या प ह या बैठक त अँ लो इं डयानाचे त न ध व ँ क अँथनी यांनी केले.

Options

a  फ अ, ब, ड
b  फ अ, ब, क
c  फ ब, क, ड
d  वरीलपैक सव

Your Answer unanswered

Correct Answer d
Explanation
 

Question No. 92
Question
वतं भारताचे प हले मं मडळ (1947) यातील नेते व संबं धत खाते यां या यो य जोडया जुळवा.

      तंभ (A)                                                                                तंभ (B)

         नेत े                                                                                         खाते.

(अ)    डॉ.जॉन मथाई                                                          (i) उदयोग व पुरवठा

(ब)    जगजीवन राम                                                              (ii) संचार

(क)   रफ अहमद कडवाई                                                    (iii) म

(ड)   डॉ.शाम साद मुखज                                                       (iv) रे वे व वाहतूक

Options

a  अ-i, ब-ii, क-iii, ड-iv


b  अ-iv, ब-iii, क-ii, ड-i
c  अ-iv, ब-ii, क-i, ड-iii
d  अ-iv, ब-i, क-iii, ड-ii

Your Answer unanswered

Correct Answer b
Explanation
 

Question No. 93
Question
भारत सरकार कायदा - 1919 बाबत खालील वधाने वचारात या.

(अ)   या कायदय्◌ा वये भारतात क ाम ये थमच दगृही कायदे मंडळाची न मती कर यात आली.

(ब)     या कायदया वये मालम ा कर कवा श ण या आधारावर मया दत लोकांना मता धकार दे यात आला.

(क)    या कायदय्◌ात लाेकसेवा आयोग थापन कर याची तरतूद होती.

(ड)    या काय ा वये थमच ांतांचे अथसंक प हे क य अथसंक पात समा व कर यात आले.

Options

a   वधान अ, ब बरोबर परंतु क, ड चूक आहेत.


b   वधान क, ड बरोबर परंतु अ, ब, चूक आहेत.
c   वधान अ,ब, क बरोबर परंतु ड चूक
d   वधाने ब, क, ड बरोबर परंतु अ चूक आहे.

Your Answer unanswered

Correct Answer c
Explanation
 

Question No. 94
Question

पुढ ल दोन वधानांपैक काेणती/ते वधान/ने बरोबर आहेत.

(अ)      भारत सरकार कायदा 1919 ने कर मालम ा, श ण यांवर आधा रत मया दत मता धकार दला.

(ब)      भारत सरकार कायदा 1935 ने मालम ा कवा श ण या आधारावर मया दत लोकांना मता धकार दला.
Options

a  फ अ यो य
b  फ ब यो य
c  अ व ब यो य
d  दो ही अयो य 

Your Answer unanswered

Correct Answer b
Explanation
 भारत सरकार कायदा 1935 ने मालम ा, कर श ण यांवर आधा रत मया दत मता धकार दला.
भारत सरकार कायदा - 1919 ने मालम ा कवा श ण या आधारावर मया दत मता धकार दला.

Question No. 95
Question

जो ा लावा.

                तंभ (I)                                                                                      तंभ (II)

             स मती                                                                                        अ य

(अ) ां तक रा यघटना स मती                                                      (i) डॉ. राज साद

(ब) कामकाज या नयमन स मती                                          (ii) सरदार पटे ल

(क) वृ प क स मती                                                                 (iii) उषानाथ सेन

(ड) नाग रक वावरील एकतध स मती                                        (iv) एस.वारदाचारी.

Options

a  अ-ii, ब-iii, क-iv, ड-i


b  अ-ii, ब-i, क-iv, ड-iii
c  अ-iii, ब-iv, क-ii, ड-i
d अ-ii, ब-i, क-iii, ड-iv
 

Your Answer unanswered

Correct Answer d
Explanation
 

Question No. 96
Question

सं था नकां या त नध या सं वधान सभेतील वेशाबाबत खालील वधाने अ यासा.

(अ)    28 ए ल 1947 रोजी पाच सं था नकां या त नध नी सं वधानसभेत वेश केला होता.

(ब)      कॅ बनेट मशन जाहीर झा यानंतर उव रत सं था नकां या त नध नी वेश केला.

(क)    परंतु भारतीय दे शातील मु लम लीग या त नध नी वेश केला नाही.

Options

a  फ अ
b  ब आ ण क
c  फ अ आ ण ब
d  वरीलपैक सव

Your Answer unanswered

Correct Answer d
Explanation
 

Question No. 97
Question
‘‘सावभाैम’’या श दाचा अथ खालीलपैक काय आहे ?

(अ)   ‘‘सावभौम दे श’’ हणजे या दे शावर कुठलाही बा भाव नसणे.

(ब)    ‘‘सावभौम’’ या श दाम ये परक य दे श मळ व या या अ धकाराचा समावेश होता.

(क)   ‘‘सावभौम’’या श दाम ये वत:चा दे श परक य श स बहाल कर याचा अ धकार अस याचाही समावेश होतो.

Options

a  फ अ
b  ब आ ण क
c  फ अ आ ण क
d  वरीलपैक सव

Your Answer unanswered

Correct Answer c
Explanation
 

Question No. 98
Question

खालील वधाने वचारात या.

(अ)    भारतीय रा यघटने या भाग iii ला ‘भारताचा मॅे नाकाटा’ हणून संबोधले जाते.

(ब)    ‘मॅ नाकाटा’ हणजे उमरावां या दबावामुळे इं लंड्चा राजा याने दलेली ह काची सनद हाेय.

(क)   काय ा या सरकार ऐवजी चे सरकार थापन करणे हे मूलभूत ह कांचे वै श य आहे.

Options

a   वधान अ, ब, आ ण क बरोबर आहेत.


b   वधान ब आ ण क बरोबर आहेत.
c   वधान अ आ ण ब बरोबर आहेत.
d   वधान अ आ ण क बरोबर आहेत.

Your Answer unanswered

Correct Answer c
Explanation
  या सरकारऐवजी कायदयाचे सरकार थापन करणे हे मूलभूत ह कांचे वै श य आहेत.

Question No. 99
Question

 ‘भारतर न, पद्म वभूषण,पद्मभूषण,पद्म ी ही रा ीय पा रतो षके घटना मक या वैध आहेत’’ असा नणय सव च यायालयाने
खालीलपैक काेण या खट यात दला ?

Options

a  बालाजी राघवन खटला - 1996


b  इं ा सहानी खटला - 1992
c  बाबूलाल व दध मुंबई रा य खटला - 1960
d  एस . आर . बो मई खटला.

Your Answer unanswered

Correct Answer a
Explanation
 भारतर न प वभूषण, पद्मभूषण, पद्म ी ही रा ीय पा रतो षके घटना मक या वैध आहेत असे सव च यायालयाने
बालाजी राघवन खट यात 1996 साली सां गतले.

Question No. 100


Question
खालील वधानांची स यता तपासा आ ण खालीलपैक काेणते वधान न त बरोबर आहे ते ओळखा.

वधान (अ) : के.ट .शहा यांनी धम नरपे व समाजवाद ही दो ही वशेषणे भारतीय संघरा याला लावावीत असे सुच वले होते.
वधान (ब) : डॉ. बाबासाहेब आंबेडकरांनी यांस वरोध क न अशी वशेषणे जास ाकाला लावणे हणजे लाेकशाही न
कर यासारखे आहे असे हटले होते.
 

Options

a   वधान अ  आ ण  ब दो हीही बरोबर आहेत. 


b   वधान अ  आ ण  ब दो हीही चूक आहेत.
c   वधान अ बरोबर असून वधान ब चूक आहे.
d वधान अ चूक असून वधान ब बरोबर आहे. 

Your Answer unanswered

Correct Answer a
Explanation
 (1) धम नरपे व समाजवाद ही वशेषणे भारतीय संघरा याला लावावीत असे सुच वले - के. ट शाह.
(2) अशी (धम नरपे व समाजवाद ) वशेषणे जास ाकाला लावणे हणजे लोकशाही न कर यासारखे आहे, असे
डॉ.बाबासाहेब आंबेडकरांनी हटले होते.

About Us
The Unique Academy is a leading Institue empowering the civil services aspirants to realise their
dreams.

Contact Us
Email : info@theuniqueacademy.co.in

Contact : 7620 44 66 44

Social Media :

   
Quick Links
About Us (about.php)

Contact Us (contact.php)

Terms and Conditions (uniterms.php)


Privacy Policy (uniprivacy.php)

Refund Policy (unirefund.php)

Disclaimer Policy (unidisclaimer.php)

(https://play.google.com/store/apps/details?

id=com.uniqueacademy.android)

(https://apps.apple.com/in/app/the-unique-
academy/id1481698588)

You might also like